Se $C \cup \{B_i\}$ não admitisse subcobertura finita, então $C \cup \{B_i\} \in \mathcal{C}$ e $C \subset C \cup \{B_i\}$, o que contradiz a hipótese de $C$ ser maximal, a menos que $\{ B_i \} \in C$. \\ Mas nesse caso $\{B_i\} \in \mathcal{B} \cap \mathcal{C}$ e $x \in B_1 \cap \cdots \cap B_n \subset A \Rightarrow x \in B_i \in \mathcal{C}$, o que contradiz a hipótese de que $x \notin B \forall B \in \mathcal B \cap C$. \\ Logo, $C \cup \{B_i\}$ tem subcobertura finita.